LSAT and Law School Admissions Forum

Get expert LSAT preparation and law school admissions advice from PowerScore Test Preparation.

 Administrator
PowerScore Staff
  • PowerScore Staff
  • Posts: 8916
  • Joined: Feb 02, 2011
|
#23853
Complete Question Explanation

Resolve the Paradox. The correct answer choice is (C)

Shoppers without a list spent more money than the shoppers with a list, even though the latter group bought only items that were on sale. This is puzzling given that on-sale items are expected to cost less. However, if only the most expensive items go on sale, then no wonder the “frugal” shoppers end up spending more. Answer choice (C) contains a possible cause of the situation and is therefore correct.

Answer choice (A): It is unclear how using a shopping cart would help shoppers with a list save money. If anything, using a shopping cart might enable shoppers to purchase more items and therefore spend more money. This answer choice is incorrect.

Answer choice (B): At first, this may seem like an extremely attractive answer choice. If shoppers without a shopping list bought many unnecessary items, wouldn’t this explain why they spent more money? Perhaps, but only if they purchased more items than the shoppers with a shopping list. However, if the two groups bought a comparable number of items (which is clearly stated in the stimulus), it is still puzzling why the first group of shoppers spent more money than the second.

Remember: the correct answer to a Resolve the Paradox question will actively resolve the paradox, that is, it will allow both sides to be factually correct while adding a piece of information that shows how they can coexist. Many incorrect answers will try to lure you with reasonable solutions that do not quite meet the stated facts. These answers are incorrect. If a given answer choice does not conform to the specifics of the stimulus, it cannot explain the situation.

Answer choice (B) also highlights the importance of reading all five the answer choices before making a decision.

Answer choice (C): This is the correct answer choice. See discussion above.

Answer choice (D): How the grocery store in the study compares to other grocery stores is irrelevant. Both groups of shoppers shopped at the same grocery store.

Answer choice (E): If there were relatively few items on sale, then the shoppers who only bought on-sale items must have been quite limited in their choice of products. It is even more puzzling that they spent more money than the other group.
 Sherry001
  • Posts: 81
  • Joined: Aug 18, 2014
|
#20067
Hello ;
I was hesitant in choosing the correct answer for this question . Could you please look over my reasoning ? I wasted so much time debating between B And C ( for the reasons below , B was very tempting for me ).!


Q TYPE : paradox . I'm looking for an answer that would explain this issue at hand while allowing both sides to remain true.

P1: on comparable number of items: shoppers who used no shopping list and bought only sell items spent more money then those who used a list and didn't buy any sell items.

A) doesn't explain

B) I saw this choice as explaining the paradox. I felt because we're told comparable number of items , (which doesn't necessarily mean 100% equal ) , and " many " as indicated in this choice could mean some? would work! What if the people who only bought sale items got 1 or 2 extra items that was though on sale, still more expensive than a regular price item ?

C) I see this explaining the paradox.

D) irrelevant
E) irrelevant
 Robert Carroll
PowerScore Staff
  • PowerScore Staff
  • Posts: 1787
  • Joined: Dec 06, 2013
|
#20077
Sherry,

The problem with answer choice (B) is that, while the first group bought "many" unnecessary items, that doesn't actually mean the items bought were in excess of the number bought by the second group. As you pointed out, we know from the stimulus that the numbers were comparable. Answer choice (B) tells us neither that the first group bought more items than the second, nor that the items bought were more expensive. Thus, it does nothing to resolve the paradox.

Robert Carroll
 Sherry001
  • Posts: 81
  • Joined: Aug 18, 2014
|
#20085
Awesome ! thank you for the clear explanation.
User avatar
 LawSchoolDream
  • Posts: 57
  • Joined: Jan 18, 2024
|
#105347
Robert Carroll wrote: Wed Sep 30, 2015 5:43 pm Sherry,

The problem with answer choice (B) is that, while the first group bought "many" unnecessary items, that doesn't actually mean the items bought were in excess of the number bought by the second group. As you pointed out, we know from the stimulus that the numbers were comparable. Answer choice (B) tells us neither that the first group bought more items than the second, nor that the items bought were more expensive. Thus, it does nothing to resolve the paradox.

Robert Carroll

Hi,

I understand this logic and it makes absolute sense however I used a reverse logic and arrived at C. I felt that even if the expensive items were on sale, doesn't mean the people that did not buy sale items didn't spend a lot. So I felt that this answer could go either way. Could you help clarify this a bit further?

For some reason when we are in the middle of solving the problem we don't see as clearly. Some answers make sense to me later but when posed a completely new question I end up using the same logic. Trying to break that.
User avatar
 Chandler H
PowerScore Staff
  • PowerScore Staff
  • Posts: 65
  • Joined: Feb 09, 2024
|
#105564
LawSchoolDream wrote: Sun Feb 18, 2024 5:27 pm
Hi,

I understand this logic and it makes absolute sense however I used a reverse logic and arrived at C. I felt that even if the expensive items were on sale, doesn't mean the people that did not buy sale items didn't spend a lot. So I felt that this answer could go either way. Could you help clarify this a bit further?

For some reason when we are in the middle of solving the problem we don't see as clearly. Some answers make sense to me later but when posed a completely new question I end up using the same logic. Trying to break that.
Hi LawSchoolDream,

Let's try to clarify things by using some numbers. Answer choice (C) tells us that "only the most expensive items go on sale in grocery stores." Let's say you and I both buy 3 items. All three of my items are on sale, so they each went down from $15 apiece to $10 apiece. Yours are not on sale, but all three of your items are priced at $5 originally.

That means that you spent $15 total, but I spent $30, even though all of my items were on sale and none of yours were. It doesn't matter how much you spent, ultimately—for example, all of your items could have been $10 originally, or even $20, but the items I bought are the most expensive items in the store, meaning they would be more expensive than your items even if they ARE on sale.

Does that make sense?

Get the most out of your LSAT Prep Plus subscription.

Analyze and track your performance with our Testing and Analytics Package.